3x+15=24 what is the x equal too

Answers

Answer 1

Answer:

x is equal to -1/3

hope this helps

Answer 2

Answer: I'm pretty sure x = 3

Step-by-step explanation: 3 times 3 = 9. 9 + 15 = 24. And when I solve for x it equals 3.


Related Questions

which number is the same as 0.000 000 1: 10^7 or 10^-7​

Answers

Answer:

= 1 × 10^-7

(scientific notation)

= 1e-7

(scientific e notation)

= 100 × 10^-9

(engineering notation)

(billionth; prefix nano- (n))

= 0.0000001

(real number)

Step-by-step explanation:

10^7 = 10000000

Please Help! :(
Six teams are out on the field playing soccer. The teams all have the same number of players. The head coach asks for 2 players from each team to come help him move some equipment. Now there are 78 players on the field. Write and solve an algebraic equation whose solution is the number of players, p, on each team.

Write the Equation and the answer please

Answers

Answer: there is 90 players in total

Step-by-step explanation:

6x 2= 12

78 + 12 = 90

Answer:

x=15

Step-by-step explanation:

6(x−2)=78 (or 6x−12=78), x=15

The sign of the leading coefficient for the polynomial equation of the graph is negative positive zero

Answers

Answer:

The sign of the leading coefficient for the polynomial equation of the graph is NEGATIVE

Hope this helps!

Answer: 1 turning point

Step-by-step explanation:

What is the area of a rectangle with sides measuring 2x+3 and 3x-1 feet

Answers

Here is your answer
the answer is 6x^2+7x-3
u would foils the two terms as shown below

a stone which is dropped into a dry well hits the bottom in 2.2s how deep is the well. take g=10m/s​

Answers

Answer:

23.7m

Step-by-step explanation:

Given parameters:

       Time taken = 2.2s

     Acceleration due to gravity = 10m/s

Unknown:

Depth of the well  = ?

Solution:

To solve this problem, we simply apply one of the motion equations;

        S = ut + [tex]\frac{1}{2}[/tex] gt²

    where S = depth of the well

                u = initial velocity

                t = time taken

               g = acceleration due to gravity

 Input the parameters and solve for S;

   Note initial velocity = 0;

     S = [tex]\frac{1}{2}[/tex] x 9.8 x 2.2²

    S = 23.7m

What is 4/9 - 1/3 =
help

Answers

1/9

Hope this helps! :))

Answer:djjdd

Please I love free points secy

Step-by-step explanation:

Ok but guys is 10009

20 points & Brainliest

Why are rotations a rigid motion?

Answers

Answer:

RC,\α is a rigid motion.

Here C, is the rotacenter which is the fixed point and around C all other points move in a fixed distance by the definition of rotation.

the motion follows in a circularmanner with C as the center.

consider a point Q which changes to Q1 after rotation by an angle\alpha

Step-by-step explanation:

When considering rotational motion and subsequently rotational mechanics every point on the body must have same angular velocity and angular acceleration. If the body is not rigid different points on the body will have different angular velocity and angular acceleration.

Herman is standing on a ladder that is partly in a hole. He starts out on a rung that is 6 feet under ground, climbs up 14 feet, then climbs down 11 feet. What is Herman’s final position, relative to ground level?

Answers

Answer:

Are there answers

Step-by-step explanation:

Answer:

answer

Herman is standing on a ladder that is partly in a hole. He starts out on a rung that is 6 feet under ground, climbs up 14 feet, then climbs down 11 feet. ...

When is it appropriate to model data with a liner function? Give an example of real world data that can be modeled with a linear function. Include the linear function and a sample of the data

Answers

Answer:

The linear function would be:

C = 10t + 2

where C is the cost and t is time in months

Step-by-step explanation:

What is the measure of angle F?
E. 20
D. 90
F.?

Answers

Answer:

the answer would be 70 because you take 90-20 and you get 70. you have a 90 degree angle is what you're working on right now

Step-by-step explanation:

On the beach boardwalk there are 20 different places to get food 20% of them are ice cream shops how many ice cream Shoppe’s are in the park

Answers

4 ice cream shops
hope this helps!

Answer:

( 4 )

Step-by-step explanation:

Hope this helps

also go to g-mail to check out my

Type in Brandon Acevedo

okay

Think of a problem that you may be interested in that deals with a comparison of two population means. Propose either a confidence interval or a hypothesis test question that compares these two means. Gather appropriate data and post your problem (without a solution) in the discussion topic. Later, respond to your own post with the solution for others to check their work. For example, you may want to know if the average weight of a rippled potato chip is the same as the average weight of a non-rippled potato chip. You may weigh rippled regular potato chips fro m a large bag and find weights of 1.7,1.9,2.4,1.3, 1.7, and 2.0 grams. You may weigh non-rippled potato chips from another larg bag and find weights of 1.8, 1.6,1.9, 1.9, and 1.4 grams. Assume a random sample was drawn. Think of a problem that you may be interested in that deals with a comparison of two population proportions. Propose either a confidence interval or a hypothesis test question that compares these two proportions. Gather appropriate data and post your problem (without a solution) in the discussion topic. Later, respond to your own post with your own solution. For example, you may believe that the proportion of adults in California who are vegetarians is more than the proportion of adults in New Hampshire who are vegetarians. In two independent polls, you may find that 109 out of 380 California residents are vegetarians and 39 out of 205 New Hampshire residents are vegetarians. For your response to a classmate (two responses required, one in each option), solve your classmate's confidence interval or hypothesis test problem, using a significance level not previously used Make sure that you use appropriate terminology and specify whether you are using the classical method or the p-value method.

Answers

Answer:

Responds to my friend

Decision rule

 Fail to reject the null hypothesis

Conclusion

There is no sufficient evidence to conclude that the proportion of residents in California who are vegetarians is higher than the proportion in New Hampshire who are vegetarian  

Step-by-step explanation:

Here we are going to consider this question

For example, you may believe that the proportion of adults in California who are vegetarians is more than the proportion of adults in New Hampshire who are vegetarians. In two independent polls, you may find that 109 out of 380 California residents are vegetarians and 39 out of 205 New Hampshire residents are vegetarians.

Here we are going to be solving the hypothesis test problem and we will be making use of the p-value method

From the question the we are told that

     The first sample size is  [tex]n_1 = 380[/tex]

       The second sample size is [tex]n_2 = 205[/tex]

       The number of California residents that are vegetarian is [tex]k = 109[/tex]

       The number of New Hampshire resident that are veterinarian is [tex]u = 39[/tex]

Generally the sample proportion for California residents is

      [tex]\^ p_1 = \frac{k}{n_1}[/tex]

=> [tex]\^ p_1 =\frac{109}{380}[/tex]

=> [tex]\^ p_1 =  0.2868 [/tex]

Generally the sample proportion for New Hampshire residents is

     [tex]\^ p_2 = \frac{39}{205}[/tex]

=>  [tex]\^ p_2 = 0.1902 [/tex]  

The null hypothesis is  [tex]H_o  :  p_1 -p_2 = 0[/tex]

The alternative hypothesis is  [tex]H_a :  p_1 -p_2 > 0[/tex]

 Generally the test statistics is mathematically represented  as

[tex]t  =  \frac{(\^ p_1 - \^ p_2 ) -0}{\sqrt{ \frac{\^ p_1 (1-\^p_1 )}{n_1}  + \frac{\^ p_2 (1- \^p_2)}{n_2} } }[/tex]

=> [tex]t  =  \frac{( 0.2868 - 0.1902) -0}{\sqrt{ \frac{0.2868 (1-0.2868 )}{380}  + \frac{0.1902(1- 0.1902)}{205} } }[/tex]  

=>  [tex]t  =  0.036[/tex]

Generally the degree of freedom is mathematically represented as

      [tex]df  =  n_1 + n_2 -2[/tex]

=>    [tex]df  =  380 + 205 -2[/tex]

=>    [tex]df  =  583/tex]

Generally the probability of  [tex]t  =  0.036[/tex] at a degree of freedom of   [tex]df  =  583/tex]  from the t- distribution table is  

      [tex]p-value  =  P(t >  0.036) =  0.48564732[/tex]

Let take the level of significance to be [tex]\alpha = 0.05[/tex]

So from the values obtained we see that  [tex]p-value  >  \alpha[/tex] hence the decision rule is  

   Fail to reject the null hypothesis

The conclusion is

  There is no sufficient evidence to conclude that the proportion of residents in California who are vegetarians is higher than the proportion in New Hampshire who are vegetarian

Use the information to find the minimum sample size required to estimate an unknown population mean µ. Margin of error: $110, confidence level: 95%, σ = $500. Group of answer choices

Answers

Answer:

The minimum sample size required is 79.

Step-by-step explanation:

The following are given in the question:

e = Margin of error = 110

σ = Standard deviation = 500

Confidence level = 95%

n = Minimum sample size = ?

Therefore, we have:

Zσ = 1.96 at 95% confidence level

s.e. = Standard error = σ / [tex]\sqrt{n}[/tex] = 500 /

Alternatively, margin of error can be calculated as follows:

e = s.e. * Zσ = 110 ................... (1)

Substituting the other values into equation (1) and solve for n, we have:

500 / [tex]\sqrt{n}[/tex] * 1.96 = 110

500 / [tex]\sqrt{n}[/tex]  = 110 / 1.96

500 / [tex]\sqrt{n}[/tex]  = 56.1224489795918

500 = 56.1224489795918 *  [tex]\sqrt{n}[/tex]  

[tex]\sqrt{n}[/tex]  = 500 / 56.1224489795918

[tex]\sqrt{n}[/tex]  = 8.90909090909092

[tex]n^{\frac{1}{2}[/tex] = 8.90909090909092

Squaring both sides, we have:

([tex]n^{\frac{1}{2}[/tex])^2 = (8.90909090909092)^2

n = 79.3719008264465

Approximating to a whole number, we have:

n = 79

Therefore, the minimum sample size required is 79.

Six and forty-two ten-thousandths as a decimal

Answers

6.0042 is the answer
6.042 is the answer hope u get this

Is f^-1(x) = x/9 - 5/9 a function?

Answers

⠀ ⠀ ⠀ ⠀ ⠀ ⠀ ⠀_____________

⠀ ⠀ ⠀ ⠀ ⠀ ⠀ ⠀;; ∥ • Given:

⠀[tex]f-¹(x)=\frac{x}{9}-\frac{5}{9}[/tex].

⠀ ⠀ ⠀ ⠀ ⠀ ⠀ ⠀_____________

⠀ ⠀ ⠀ ⠀ ⠀ ⠀ ⠀;; ∥ • To find out:

⠀Is a function [tex]f-¹(x)=\frac{x}{9}-\frac{5}{9}[/tex] or not.

⠀ ⠀ ⠀ ⠀ ⠀ ⠀ ⠀_____________

⠀ ⠀ ⠀ ⠀ ⠀ ⠀ ⠀;; ∥ • Hello, ZoFlo2020!

⠀ ⠀ ⠀ ⠀ ⠀ ⠀ ⠀_____________

⠀ ⠀ ⠀ ⠀ ⠀ ⠀ ⠀;; ∥ • Answer (Response Explanation):

⠀ ⠀ ⠀ ⠀ ⠀ ⠀ ⠀_____________

⠀No, it is not. Such an example is not even possible to solve, to be honest. It doesn’t have a single solution to find at least an approximate part of its solution and answer.

⠀ ⠀ ⠀ ⠀ ⠀ ⠀ ⠀_____________

⠀ ⠀ ⠀ ⠀ ⠀ ⠀ ⠀;; ∥ • Sincerely, Your ElskerNorge! :)

⠀ ⠀ ⠀ ⠀ ⠀ ⠀ ⠀_____________

Which of these is the triangle proportionality theorem? First person to answer CORRECTLY gets brainliest

Answers

Answer: A.

Step-by-step explanation: Brainliest plz

PLEASE ANSWER I WILL GIVE YOU BRAINLIEST!
Name the points which satisfy the conditions. In the second quadrant

Answers

9514 1404 393

Answer:

  C, M

Step-by-step explanation:

The second quadrant is the upper left quadrant. There, x-values will be negative and y-values will be positive.

__

Point G is on the boundary with the 3rd quadrant, so is probably not considered to be in either of those quadrants.

Answer: C M

Step-by-step explanation:

HELP ME PLEASE IM ALMOST TO 100

Answers

Answer:

= 3/4

Step-by-step explanation:

Find 2 points on the graph

(2,3) and (6,6)

Using the slope formula

m = (y2-y1)/(x2-x1)

   = ( 6-3)/(6-2)

   = 3/4

(4.3 x 10^-3)(2 x 10^-2) in standard form

Answers

Answer:

8.6•10^-5 or 0.000086

if there are 7 fish and 3 drown how many are left?​

Answers

Answer:

7

Step-by-step explanation:

there are 7 left because fish can't drown

Fish can’t drown lol if this was a joke or test.

what is 4/6+2/3.?(these are fractions, you're just adding fractions)

Answers

Answer:

4/3

Step-by-step explanation:

Multiply 2 by 2, and get 4, then we multiply 3 by 2 and get 6.

Since our denominators match, we can add the numerators. 4 + 4 = 8. That gives us the sum, which is 8/6. Then you divide by 2.  So 8/6 / 2 gives you 4/3.

Answer:

4/3 or 1 1/3

Step-by-step explanation:

1. 4/6 + 2/3

2. 4/6 + 4/6          You have to find the LCM (least common multiple) of the two  denominators. In this case it would be 6. You have to multiply 3 by 2 to get 6. What you do to the bottom you also do to the top so you have to also multiply the numerator (2) by 2.

3. 4/3 or 1 1/3       Afterward, you just have to add straight across without changing the denominator and then if you need to simplify you simplify. Without simplifying, you would get 8/6, but, most teachers will mark it as wrong if you don't simplify. You can then change the improper fraction into a mixed number depending on what your teacher wants.

Identify the coefficients: -3h - 2h + 6h +9

Answers

Answer:

-23h

Step-by-step explanation:

It’s -3, 2, 6 is the answer

Quickkkk!!!!!!!! Help

Answers

Answer:

2143

Step-by-step explanation:

A car sharing service offers a membership plan with a $50 per month fee that includes 10 hours of driving each month and charges $9 for each additional hour
(A) Write a piecewise definition of the cost F(x) for a month in which a member uses a car for x hours
(B) Graph F(x) for 0 < x s 15
(C) Find lim x--->10 F(x), lim x--->10F(x), and lim x--->10F(x), which- ever exist.

Answers

Answer:

A)  F(x) = 50 + ( x - 10 )*9  

B) Graph

C) lim ( x ⇒ 10) F(x) = 50

Step-by-step explanation:

A) F(x) = 50 + ( x - 10 )*9       since the first 10 hours are included in initial fee

B) In Annex

C) lim (x ⇒ 10) F(x) = 50 + ( 10 -10)*9

lim ( x ⇒ 10) F(x) = 50 + 0

lim ( x ⇒ 10) F(x) = 50

What is the answer tho

Answers

Answer:

to?

Step-by-step explanation:

uh what’s ur question?

if loga81=4, find value of a?​

Answers

Answer:

[tex]\huge\boxed{a=4}[/tex]

Step-by-step explanation:

[tex]\log_xy=z\iff x^z=y\\\\\text{for}\ x\neq1\ \wedge\ x>0\ \wedge\ y>0\\==================\\\\\log_a81=4\iff a^4=81\\\\a^4=\underbrace{3\cdot3\cdot3\cdot3}_{=81}\\\\a^4=3^4\Rightarrow a=3[/tex]

A TV show has 26 episodes. Due to the lack of time, you can onlywatch 5 of them. However, in order to get a gist of what the showis about, you do not want to watch any two consecutive episodes.How many selections of 5 episodes are there with no two consecutiveepisodes chosen? Assuming that the order of selection is irrelevant.(Hint:this is a Stars and Bars problem)

Answers

Answer:

Step-by-step explanation:

Recall that: The number of combination to select any k consecutive element from n consecutive term is given by the equation [tex]^{n-k+1}C_k[/tex]

[tex]= \begin {pmatrix} ^{n-k+1}_{k} \end {pmatrix}[/tex]

where:

n = 26

k = 5

[tex]^{n-k+1}C_k= \begin {pmatrix} ^{26-5+1}_{5} \end {pmatrix}[/tex]

[tex]^{n-k+1}C_k= \begin {pmatrix} ^{22}_{5} \end {pmatrix}[/tex]

[tex]^{n-k+1}C_k= \dfrac{22!}{5!(22-5)!}[/tex]

[tex]^{n-k+1}C_k= \dfrac{22!}{5!(17)!}[/tex]

[tex]\mathbf{^{n-k+1}C_k=26334}[/tex]

Therefore, there are 26334 selection of 5 episodes with no two consecutive episodes chosen

Solve for j if aj+k=v

Answers

Answer:

j=v-k/a

Step-by-step explanation:

It's like order of operations backwards. So first, you would get rid of the k, by subtracting it from both sides. Then, to isolate j you would divide both sides by a.

13
In the figure shown below, if line mis parallel to line n, then find the value of x. (This is a 2 part question.)
m
(2x-10)
(65 - x)
Part A
Write the equation to find the value of x. (DO NOT find x, just write the equation.)
(2x - 10) (65 - x)
Part B
What is the value of x?
Y

Answers

Answer:

2x-10 = 65-x

Step-by-step explanation:

just add up both alternate internal angles as they are both equal

so, the equation is: 2x-10 = 65-x

Answer:

The right equation will be 2x - 10 = 65 - x (you were wrong lol)

Step-by-step explanation:

We have: 2x - 10 = 65 - x

=> 2x - 10 + x = 65

=> 3x = 65 + 10 = 75

=> x = 75/3 = 25

P/S: After some little chats with the others, this might be wrong :(

One of the dog treats in Shelley’s shop is 8 inches long and 3.2 inches wide. She wants to make a miniature version that has proportional dimensions. If the miniature treat will be 3 inches long, how wide should it be?

Answers

Answer:

1.2in

Step-by-step explanation:

Answer:

(1) 1.2 inches

Other Questions
Draw diagonals ACand BDon rectangle ABCD. Measure and record the lengths of the diagonals. Can somebody help me with this question really quick? I think it may be c or d but Im not sure Adams began to worry that members of his party were willing to use force to destroy who someone please help! Ill give brainliest!! If A is the least common multiple of 12, 24, and 36, and B is the lowest prime number, what is the sum of A and B ?F. 3G. 4H. 73J. 74K.145 [(2r^2t)^3/4t^2]^2 simply what do reflections have that dilation do not dose anybody know this? could you provide the step by step Find the values of x and y. x= I WILL GIVE YOU BRAINLIEST!!!Read the poem "We Real Cool" by Gwendolyn Brooks.The Pool Players.Seven at the Golden Shovel.We real cool. WeLeft school. WeLurk late. WeStrike straight. WeSing sin. WeThin gin. WeJazz June. WeDie soon.Which tone does the language in the final lines of "We Real Cool" create?serioussarcastichopefuluncertain Make a table for the function. You will be using your table to graph on the next pro y - 2x + 3 Domain:{-3, -1, 0, 2} y - 1 0 2 What is the factorization of the expression below?25x^2 - 36A. (5x+6)(5x-6)B. (5x-6)(5x-6)C. (5x- 4)(5x-9)D. (5x + 4)(5x-9) How was the Gag Rule used in the House of Representatives? Which answer choice correctly describes how the US Constitution fixed a problem or problems of the Articles of Confederation?A Under the Constitution, the people could directly elect their representatives in the US Senate.B Under the Constitution, the state governments were granted more control over immigration and trade.C Under the Constitution, the federal government had the power to enforce laws but was checked by Congress and the court system.D Under the Constitution, political candidates must be a member of a political party so that they represent the interests of the people. Select each polynomial expression that could be factored by grouping. 3x + 2y + z = 7 2x + 3y - z = 8 5x + y + 2z = 11 the first generation of computers used vaccum tubes .Why were they replaced by the transistors What is 40 percent of 50? 1. 202. 303. 404. 10 Look at the table. Bob ran 3 miles in 18 minutes. Complete the table: determine the range of the following graph: